Stay ahead of learning milestones! Enroll in a class over the summer!

G
Topic
First Poster
Last Poster
Problem 1
SpectralS   145
N 6 minutes ago by IndexLibrorumProhibitorum
Given triangle $ABC$ the point $J$ is the centre of the excircle opposite the vertex $A.$ This excircle is tangent to the side $BC$ at $M$, and to the lines $AB$ and $AC$ at $K$ and $L$, respectively. The lines $LM$ and $BJ$ meet at $F$, and the lines $KM$ and $CJ$ meet at $G.$ Let $S$ be the point of intersection of the lines $AF$ and $BC$, and let $T$ be the point of intersection of the lines $AG$ and $BC.$ Prove that $M$ is the midpoint of $ST.$

(The excircle of $ABC$ opposite the vertex $A$ is the circle that is tangent to the line segment $BC$, to the ray $AB$ beyond $B$, and to the ray $AC$ beyond $C$.)

Proposed by Evangelos Psychas, Greece
145 replies
SpectralS
Jul 10, 2012
IndexLibrorumProhibitorum
6 minutes ago
Integrable function: + and - on every subinterval.
SPQ   3
N Today at 7:06 AM by solyaris
Provide a function integrable on [a, b] such that f takes on positive and negative values on every subinterval (c, d) of [a, b]. Prove your function satisfies both conditions.
3 replies
SPQ
Today at 2:40 AM
solyaris
Today at 7:06 AM
Putnam 1999 A4
djmathman   7
N Today at 7:05 AM by P162008
Sum the series \[\sum_{m=1}^\infty\sum_{n=1}^\infty\dfrac{m^2n}{3^m(n3^m+m3^n)}.\]
7 replies
djmathman
Dec 22, 2012
P162008
Today at 7:05 AM
Find the greatest possible value of the expression
BEHZOD_UZ   1
N Today at 6:34 AM by alexheinis
Source: Yandex Uzbekistan Coding and Math Contest 2025
Let $a, b, c, d$ be complex numbers with $|a| \le 1, |b| \le 1, |c| \le 1, |d| \le 1$. Find the greatest possible value of the expression $$|ac+ad+bc-bd|.$$
1 reply
BEHZOD_UZ
Yesterday at 11:56 AM
alexheinis
Today at 6:34 AM
Problem vith lcm
snowhite   2
N Today at 6:21 AM by snowhite
Prove that $\underset{n\to \infty }{\mathop{\lim }}\,\sqrt[n]{lcm(1,2,3,...,n)}=e$
Please help me! Thank you!
2 replies
snowhite
Today at 5:19 AM
snowhite
Today at 6:21 AM
combinatorics
Hello_Kitty   2
N Yesterday at 10:23 PM by Hello_Kitty
How many $100$ digit numbers are there
- not including the sequence $123$ ?
- not including the sequences $123$ and $231$ ?
2 replies
Hello_Kitty
Apr 17, 2025
Hello_Kitty
Yesterday at 10:23 PM
Sequence of functions
Squeeze   2
N Yesterday at 10:22 PM by Hello_Kitty
Q) let $f_n:[-1,1)\to\mathbb{R}$ and $f_n(x)=x^{n}$ then is this uniformly convergence on $(0,1)$ comment on uniformly convergence on $[0,1]$ where in general it is should be uniformly convergence.

My I am trying with some contradicton method like chose $\epsilon=1$ and trying to solve$|f_n(a)-f(a)|<\epsilon=1$
Next take a in (0,1) and chose a= 2^1/N but not solution
How to solve like this way help.
2 replies
Squeeze
Apr 18, 2025
Hello_Kitty
Yesterday at 10:22 PM
A in M2(prime), A=B^2 and det(B)=p^2
jasperE3   1
N Yesterday at 9:59 PM by KAME06
Source: VJIMC 2012 1.2
Determine all $2\times2$ integer matrices $A$ having the following properties:

$1.$ the entries of $A$ are (positive) prime numbers,
$2.$ there exists a $2\times2$ integer matrix $B$ such that $A=B^2$ and the determinant of $B$ is the square of a prime number.
1 reply
jasperE3
May 31, 2021
KAME06
Yesterday at 9:59 PM
Equation over a finite field
loup blanc   1
N Yesterday at 9:30 PM by alexheinis
Find the set of $x\in\mathbb{F}_{5^5}$ such that the equation in the unknown $y\in \mathbb{F}_{5^5}$:
$x^3y+y^3+x=0$ admits $3$ roots: $a,a,b$ s.t. $a\not=b$.
1 reply
loup blanc
Yesterday at 6:08 PM
alexheinis
Yesterday at 9:30 PM
Integration Bee Kaizo
Calcul8er   51
N Yesterday at 7:41 PM by BaidenMan
Hey integration fans. I decided to collate some of my favourite and most evil integrals I've written into one big integration bee problem set. I've been entering integration bees since 2017 and I've been really getting hands on with the writing side of things over the last couple of years. I hope you'll enjoy!
51 replies
Calcul8er
Mar 2, 2025
BaidenMan
Yesterday at 7:41 PM
interesting integral
Martin.s   1
N Yesterday at 2:46 PM by ysharifi
$$\int_0^\infty \frac{\sinh(t)}{t \cosh^3(t)} dt$$
1 reply
Martin.s
Monday at 3:12 PM
ysharifi
Yesterday at 2:46 PM
postaffteff
JetFire008   19
N Apr 15, 2025 by JetFire008
Source: Internet
Let $P$ be the Fermat point of a $\triangle ABC$. Prove that the Euler line of the triangles $PAB$, $PBC$, $PCA$ are concurrent and the point of concurrence is $G$, the centroid of $\triangle ABC$.
19 replies
JetFire008
Mar 15, 2025
JetFire008
Apr 15, 2025
postaffteff
G H J
G H BBookmark kLocked kLocked NReply
Source: Internet
The post below has been deleted. Click to close.
This post has been deleted. Click here to see post.
JetFire008
124 posts
#1
Y by
Let $P$ be the Fermat point of a $\triangle ABC$. Prove that the Euler line of the triangles $PAB$, $PBC$, $PCA$ are concurrent and the point of concurrence is $G$, the centroid of $\triangle ABC$.
Z K Y
The post below has been deleted. Click to close.
This post has been deleted. Click here to see post.
JetFire008
124 posts
#2
Y by
The Fermat Point of a triangle is the interior point from which the sum of distance between vertices is minimum.
Z K Y
The post below has been deleted. Click to close.
This post has been deleted. Click here to see post.
JetFire008
124 posts
#3
Y by
Euler line is the straight line passing through the orthocenter, centroid, and circumcenter of a triangle.
Z K Y
The post below has been deleted. Click to close.
This post has been deleted. Click here to see post.
JetFire008
124 posts
#4
Y by
Orthocentre is the point of intersection of altitudes
Centroid is the point of intersection of medians.
Circumcentre is the point of intersection of perpendicular bisectors.
Z K Y
The post below has been deleted. Click to close.
This post has been deleted. Click here to see post.
JetFire008
124 posts
#5
Y by
If the Euler line of a $\triangle ABC$ is parallel to $BC$, show that tan $B$ tan $C = 3$.
Z K Y
The post below has been deleted. Click to close.
This post has been deleted. Click here to see post.
drmzjoseph
445 posts
#6
Y by
Old problem
Extend $PA$ until $X$ such that $BXC$ is equilateral now, taking as homothetic center the midpoint of $BC$ (3:1) sending X to circumcenter of $PBC$, P to centroid of $PBC$ and A to centroid of $ABC$ that's enough
Z K Y
The post below has been deleted. Click to close.
This post has been deleted. Click here to see post.
JetFire008
124 posts
#7
Y by
Prove that the circumcircles of the four triangles formed by four lines have a common point.
Z K Y
The post below has been deleted. Click to close.
This post has been deleted. Click here to see post.
JetFire008
124 posts
#8
Y by
In $\triangle ABC$, $BD$ and $CE$ are the bisectors of $\angle B$, $\angle C$ cutting $CA$, $AB$ at $D$, $E$ respectively. If $\angle BDE = 24^{\circ}$ and $\angle CED = 18^{\circ}$, find the angles of $\triangle ABC$
This post has been edited 1 time. Last edited by JetFire008, Mar 17, 2025, 4:02 PM
Z K Y
The post below has been deleted. Click to close.
This post has been deleted. Click here to see post.
drago.7437
62 posts
#9
Y by
JetFire008 wrote:
In $\triangle ABC$, $BD$ and $CE$ are the bisectors of $\angle B$, $\angle C$ cutting $CA$, $AB$ at $D$, $E$ respectively. If $\angle BDE = 24^{\circ}$ and $\angle CED = 18^{\circ}$, find the angles of $\triangle ABC$
https://artofproblemsolving.com/community/c6h220396p1222521 here
This post has been edited 1 time. Last edited by drago.7437, Mar 18, 2025, 2:24 AM
Z K Y
The post below has been deleted. Click to close.
This post has been deleted. Click here to see post.
drago.7437
62 posts
#10
Y by
JetFire008 wrote:
Prove that the circumcircles of the four triangles formed by four lines have a common point.

Miquel
Z K Y
The post below has been deleted. Click to close.
This post has been deleted. Click here to see post.
JetFire008
124 posts
#11
Y by
Let $\triangle ABC$, $D$ be the midpoint of $BC$. Prove that
$$AB^2+AC^2=2AD^2+2DC^2$$.
Or in other words, prove the Apollonius Theorem.
This post has been edited 1 time. Last edited by JetFire008, Mar 18, 2025, 12:59 PM
Z K Y
The post below has been deleted. Click to close.
This post has been deleted. Click here to see post.
JetFire008
124 posts
#12
Y by
In $\triangle ABC$, $O$ is the circumcentre and $H$ is the orthocentre. Then, prove that $AH^2+BC^2=4AO^2$.
Z K Y
The post below has been deleted. Click to close.
This post has been deleted. Click here to see post.
JetFire008
124 posts
#13
Y by
$P$ and $P'$ are points on the circumcircle of $\triangle ABC$ such that $PP'$ is parallel to $BC$. Prove that $P'A$ is perpendicular to the Simson line of $P$.
Z K Y
The post below has been deleted. Click to close.
This post has been deleted. Click here to see post.
JetFire008
124 posts
#14
Y by
The vertices of a triangle are on three straight lines which diverge from a point, and the sides are in fixed directions; find the locus of the center of the circumscribed circle.
Source:- Problems & Solutions In Euclidean Geometry written by M.N. Aref and William Wernick
Z K Y
The post below has been deleted. Click to close.
This post has been deleted. Click here to see post.
Korean_fish_Kaohsiung
30 posts
#15
Y by
JetFire008 wrote:
Let $P$ be the Fermat point of a $\triangle ABC$. Prove that the Euler line of the triangles $PAB$, $PBC$, $PCA$ are concurrent and the point of concurrence is $G$, the centroid of $\triangle ABC$.

If we don't have to show the point is $G$ then the problem is trivial by Liang-Zelich

Click to reveal hidden text

However we will show it is $G$ , consider the midpoint of $AC$, $PC$, as $M_B, M_P$ ,let the centroid of $PBC$ be $G_A$ then we know $\dfrac{BM_B}{BG}=\dfrac{BM_P}{BG_A}$ so $GG_A$ is parallel to $M_BM_P$ which is also parallel to $AP$. now $AP$ meets the point outside of $BC$, as $P_1$ such that $BP_1C$ is an equilateral triangle, and now by $\dfrac{P_1O_A}{O_AM_A}=\dfrac{M_AG}{GA}$, where $M_A$ is the midpoint of $BC$ we have $GO_A$ is parallel to $GG_A$ so $G_A $ lies on $GO_A$ therefore it's done
Z K Y
The post below has been deleted. Click to close.
This post has been deleted. Click here to see post.
JetFire008
124 posts
#16
Y by
Let $ABC$ be an acute triangle whose incircle touches sides $AC$ and $AB$ at $E$ and $F$, respectively. Let the angle bisectors of $\angle ABC$ and $\angle ACB$ meet $EF$ at $X$ and $Y$, respectively, and let the midpoint of $BC$ be $Z$. Show that $XYZ$ is equilateral if and only if $\angle A = 60^{\circ}$.
Z K Y
The post below has been deleted. Click to close.
This post has been deleted. Click here to see post.
JetFire008
124 posts
#18
Y by
If from a point $O, OD, OE, OF$ are drawn perpendicular to the sides $BC, CA, AB$ respectively of $\triangle ABC$ then prove that
$$BD^2-DC^2+CE^2-EA^2+AF^2-FB^2=0$$Not been able to solve this even though I know I can do this.
Z K Y
The post below has been deleted. Click to close.
This post has been deleted. Click here to see post.
Captainscrubz
57 posts
#19
Y by
JetFire008 wrote:
If the Euler line of a $\triangle ABC$ is parallel to $BC$, show that tan $B$ tan $C = 3$.

Bro wants to post every problem in one Topic :stretcher: but nvm
Let $H$ be the orthocenter of $\triangle ABC$ and let $O$ be the circumcenter
Let the $\perp$ from $H$ and $O$ be $D$ and $M$
see that $HD=OM$
$\implies HD=2RcosCcosB=OM=RcosA$ then just use $cosA=-cos(B+C)$
Z K Y
The post below has been deleted. Click to close.
This post has been deleted. Click here to see post.
Captainscrubz
57 posts
#20
Y by
JetFire008 wrote:
In $\triangle ABC$, $O$ is the circumcentre and $H$ is the orthocentre. Then, prove that $AH^2+BC^2=4AO^2$.

Trigonometry bash or simply let $C'$ be the antipode and then $AC'BH$ is a rhombus
Z K Y
The post below has been deleted. Click to close.
This post has been deleted. Click here to see post.
JetFire008
124 posts
#21
Y by
Captainscrubz wrote:
JetFire008 wrote:
If the Euler line of a $\triangle ABC$ is parallel to $BC$, show that tan $B$ tan $C = 3$.

Bro wants to post every problem in one Topic :stretcher: but nvm
Let $H$ be the orthocenter of $\triangle ABC$ and let $O$ be the circumcenter
Let the $\perp$ from $H$ and $O$ be $D$ and $M$
see that $HD=OM$
$\implies HD=2RcosCcosB=OM=RcosA$ then just use $cosA=-cos(B+C)$

Did it so more people bump to this post
Z K Y
N Quick Reply
G
H
=
a